The passage most strongly supports the inference that Garber would agree with which one of the following statements?

alliehall21 on July 19, 2020

Why not E

I understand why the correct answer is B but I would like to know why E is wrong

Reply
Create a free account to read and take part in forum discussions.

Already have an account? log in

shunhe on July 20, 2020

Hi @alliehall21,

Thanks for the question! So this question is asking us for an inference about a statement that Garber would agree with. Remember, the statement can’t be something that Garber would agree in part with; Garber has to agree with the whole statement.

So let’s look at (E). It tells us that Garber would agree with the following: Mackay mistakenly infers from the fact that tulip prices dropped rapidly that the very low prices that the bulbs eventually sold for were irrational.

So what does Garber say exactly? Well, let’s look at the last paragraph, since that’s where we find out the most about Garber’s thoughts. Garber does say that “this does not mean that the high prices of original bulbs are irrational” in lines 47-49. But does Garber ever say anything about Mackay mistakenly inferring anything about low prices being irrational? No, that doesn’t appear anywhere. So because nothing is said about the irrationality of the low prices, (E) isn’t something that we can infer that Garber agrees with, and is not the correct answer.

Hope this helps! Feel free to ask any other questions that you might have.